Search found 14 matches


My pick E...whats the OA???

by sumiwali

Fri Oct 10, 2008 7:35 pm
Forum: Sentence Correction
Topic: Health Care
Replies: 7
Views: 3001

i concur with C...

my explanantion
A - has dangling modifier problem
B - SVA rural migrant worker and they do not agree.....
C- Correct
D- 'they' cannot refer to the object of the previous clause.....
E- 'they' has no clear referant.....

by sumiwali

Fri Oct 10, 2008 7:31 pm
Forum: Sentence Correction
Topic: SC: accessing / accessement
Replies: 6
Views: 2299

I agree with C .whats the OA??/

by sumiwali

Fri Oct 10, 2008 2:37 am
Forum: Sentence Correction
Topic: Qn - 857 whether vs that?
Replies: 12
Views: 1992

i think answer should be C as first equations tells us that x is an integer, else we cannot assume the same for statement 2.

whats the OA??

by sumiwali

Fri Oct 10, 2008 2:30 am
Forum: Data Sufficiency
Topic: please help ...OA needed urgently
Replies: 6
Views: 1546

i think answer should be C as first equations tells us that x is an integer, else we cannot assume the same for statement 2.

whats the OA??

by sumiwali

Fri Oct 10, 2008 2:29 am
Forum: Data Sufficiency
Topic: please help ...OA needed urgently
Replies: 6
Views: 1546

Answer B Statement1 2n has 4 different prime factors. 2n could be 2*3*5*7(i.e. for n 3 diff prime factors) OR 2n could be 2*2*3*5*7(i.e. 4 different prime factors for 2n and 4 different prime factors for n) Thus INSUFF Statement 2 n^2 has 4 different prime factors i.e.(2*3*5*7)^2 thus n will always ...

by sumiwali

Wed Oct 08, 2008 3:34 am
Forum: Data Sufficiency
Topic: GPREP
Replies: 1
Views: 1459

Lets us say he has an income x and he saves z fraction of it, thus saving is (zx) so saving would be x-zx=x(1-z) now amount available for spending next year would be (1+r)zx. this amount should be half of this years spending thus equating the two we get 0.5x(1-z)=zx(1+r) we need to solve for z(i.e. ...

by sumiwali

Wed Oct 01, 2008 5:58 am
Forum: Problem Solving
Topic: tuf ques
Replies: 1
Views: 1471

Answer Should be E.... Lets look at equation 2 to start with.... it talks about only about x intercept nothing about y ...thus NOT SUFFICIENT. For Statement 1 we know that slope m=(y interecpt)/(x intercept) and m is given to be 3 times y intercept thus 3y=y/x thus x=1/3 or it could also be -1/3 whi...

by sumiwali

Wed Oct 01, 2008 5:34 am
Forum: Data Sufficiency
Topic: DS2
Replies: 2
Views: 1441

Answer Should be E.... Lets look at equation 2 to start with.... it talks about only about x intercept nothing about y ...thus NOT SUFFICIENT. For Statement 1 we know that slope m=(y interecpt)/(x intercept) and m is given to be 3 times y intercept thus 3y=y/x thus x=1/3 or it could also be -1/3 whi...

by sumiwali

Wed Oct 01, 2008 5:23 am
Forum: Data Sufficiency
Topic: DS2
Replies: 2
Views: 1441

also when one end of the line(say PtA) lies on another line, they are said to be intersecting at PtA, thus if we have vertex of the triangle(i.e say PointX) on a circle.The triangle & cirlce would said be intersecting at the above mentioned Point B.

by sumiwali

Wed Oct 01, 2008 4:29 am
Forum: Data Sufficiency
Topic: gmat prep question
Replies: 4
Views: 1407
by sumiwali

Wed Oct 01, 2008 2:21 am
Forum: Sentence Correction
Topic: In hoping to restrain economic growth
Replies: 4
Views: 2681

I think it should be C, whats the OA??

by sumiwali

Wed Oct 01, 2008 2:19 am
Forum: Sentence Correction
Topic: George Washington Carver
Replies: 4
Views: 5417

I think it should be C, whats the OA??

by sumiwali

Wed Oct 01, 2008 2:17 am
Forum: Sentence Correction
Topic: George Washington Carver
Replies: 4
Views: 5417

i am just a beginner, so dont know if my explanation holds, but i think that the area would be the multiplication of diagonals only in the case of rhombus.

Which is not given...thus either statement which are supplying the same info are insuff and thus E. :D

by sumiwali

Sat Sep 27, 2008 1:29 am
Forum: Data Sufficiency
Topic: Princeton GMAT CAT
Replies: 1
Views: 3962